Number Theory: Structures, Examples, and Problems


I Fundamentals, 9. Some Special Problems in Number Theory



Download 1,87 Mb.
Pdf ko'rish
bet67/125
Sana08.02.2022
Hajmi1,87 Mb.
#434761
1   ...   63   64   65   66   67   68   69   70   ...   125
Bog'liq
Titu Andreescu, Dorin Andrica Number Theory Str

I Fundamentals, 9. Some Special Problems in Number Theory
If
p
=
4
u
+
1, then
n
=
2
u

u
=
u
and
p
2

1
8
=
2
u
2
+
u
. We have
n

p
2

1
8
(
mod 2
)
and we are done.
If
p
=
4
v
+
3, then
n
=
2
v
+
1

v
=
v
+
1 and
p
2

1
8
=
2
v
2
+
3
v
+
1, and
again
n

p
2

1
8
(
mod 2
)
.
The central result concerning the Legendre symbol is the so-called
quadratic
reciprocity law
of Gauss:
Theorem 9.1.3.
If p and q are distinct odd primes, then
q
p
p
q
=
(

1
)
p

1
2
·
q

1
2
.
Proof.
In Gauss’s lemma we take
a
=
q
and we get
q
p
=
(

1
)
n
. Let
"
m
i
=
1
b
i
=
b
and
"
n
j
=
1
c
j
=
c
. Then using the equality
{
b
1
, . . . ,
b
m
,
p

c
1
, . . . ,
p

c
n
} =
1
,
2
, . . . ,
p

1
2
,
it follows that
b
+
np

c
=
p

1
2
k
=
1
k
=
p
2

1
8
.
But from Gauss’s lemma we have
q
k
=
kq
p
,
k
=
1
,
2
, . . . ,
p

1; hence
q
p
2

1
8
=
p
p

1
2
k
=
1
%
kq
p
&
+
b
+
c
.
Summing up the last two relations gives
2
c
+
p
p

1
2
k
=
1
%
kq
p
&
+
p
2

1
8
(
1

q
)

np
=
0
.
Because 2
c
and 1

q
are even, it follows that
n

p

1
2
k
=
1
%
kq
p
&
(
mod 2
),
and applying Gauss’s lemma again we obtain
q
p
=
(

1
)
"
p

1
2
k
=
1
#
kq
p
$
.


9.1. Quadratic Residues; the Legendre Symbol
171
Similarly, we derive the relation
p
q
=
(

1
)
"
p

1
2
j
=
1
#
j p
q
$
.
Multiplying the last two equalities and taking into account Landau’s identity
in Problem 3.2.5(b) of Chapter 3, the conclusion follows.
Problem 9.1.1.
Let k
=
2
2
n
+
1
for some positive integer n. Show that k is a prime
if and only if k is a factor of
3
(
k

1
)/
2
+
1
.
(1997 Taiwanese Mathematical Olympiad)
Solution.
Suppose
k
is a factor of 3
(
k

1
)/
2
+
1. This is equivalent to 3
(
k

1
)/
2


1
(
mod
k
)
. Hence 3
k

1

1
(
mod
k
)
. Let
d
be the order of 3 mod
k
. Then
d
(
k

1
)/
2
=
2
2
n

1, but
d
|
(
k

1
)
=
2
2
n
. Hence
d
=
2
2
n
=
k

1. Therefore
k
is prime.
Conversely, suppose
k
is prime. By the quadratic reciprocity law,
3
k
=
k
3
=
2
3
= −
1
.
By Euler’s criterion, 3
(
k

1
)/
2

3
k
≡ −
1
(
mod
k
)
, as claimed.
Problem 9.1.2.
Prove that if n is a positive integer such that the equation x
3

3
x y
2
+
y
3
=
n has an integer solution
(
x
,
y
)
, then it has at least three such
solutions.
Show there are no solutions if n
=
2891
.
(23rd International Mathematical Olympiad)
Solution.
The idea of the solution is to find a nonsingular change of coordinates
with integer coefficients
(
x
,
y
)

(
ax
+
by
,
cx
+
d y
)
such that the polynomial
x
3

3
x y
2
+
y
3
does not change after changing coordi-
nates. Such a transformation can be found after noticing the identity
x
3

3
x y
2
+
y
3
=
(
y

x
)
3

3
x
2
y
+
2
x
3
=
(
y

x
)
3

3
(
y

x
)
x
2
+
(

x
)
3
.
Thus, such a transformation is
T
(
x
,
y
)
=
(
y

x
,

x
)
. It can be represented
by the linear transformation
T
x
y
=

1 1

1 0
x
y
=

x
+
y

x
.


172
I Fundamentals, 9. Some Special Problems in Number Theory
We have
T
2
=

1 1

1 0

1 1

1 0
=
0

1
1

1
and
T
3
=
0

1
1

1

1 1

1 0
=
1 0
0 1
.
Thus,
T
2
(
x
,
y
)
=
(

y
,
x

y
)
. Moreover, it is easy to see that if
x
3

3
x y
2
+
y
3
=
n
,
n

0, then the pairs
(
x
,
y
)
,
(

y
,
x

y
)
, and
(
y

x
,

x
)
are distinct.
For the second part, observe that 2891
=
7
2
·
59. Suppose that
x
,
y
are integers
such that
x
3

3
x y
2
+
y
3
=
2891. Then
x
,
y
are relatively prime, because from
d
=
gcd
(
x
,
y
)
we obtain
d
3
|
2891. The numbers
x
,
y
are not divisible by 7; thus
they are invertible modulo 7. Thus, from the equation we obtain
y
x
3

3
y
x
2
+
1

0
(
mod 7
).
This proves that the congruence
a
3

3
a
2
+
1

0
(
mod 7
)
has a solution,
a

Z
. Since 7 is not a divisor of
a
, by Fermat’s little theorem one
has
a
6

1
(
mod 7
)
. There are two possibilities:
a
3

1
(
mod 7
)
or
a
3
≡ −
1
(
mod 7
)
. When
a
3

1
(
mod 7
)
we obtain
a
3

3
a
2
+
1

0
(
mod 7
)

3
a
2

2
(
mod 7
)

a
2

3
(
mod 7
).
Using the Legendre symbol and the quadratic reciprocity law,
3
7
=
(

1
)
3

1
2
·
7

1
2
7
3
=
(

1
)
1
3
= −
1
.
This proves that 3 is not a square modulo 7. When
a
3
≡ −
1
(
mod 7
)
we
obtain the contradiction from 3
a
2

0
(
mod 7
)
. Thus, the equation
x
3

3
x y
2
+
y
3
=
2891 has no solution in integers
(
x
,
y
)
.
Problem 9.1.3.
Let m
,
n be positive integers such that
A
=
(
m
+
3
)
n
+
1
3
m
is an integer. Prove that A is odd.
(1998 Bulgarian Mathematical Olympiad)
Solution.
If
m
is odd, then
(
m
+
3
)
n
+
1 is odd and
A
is odd. Now we suppose
that
m
is even. Since
A
is an integer,
0

(
m
+
3
)
n
+
1

m
n
+
1
(
mod 3
),


9.1. Quadratic Residues; the Legendre Symbol
173
so
n
=
2
k
+
1 is odd and
m
≡ −
1
(
mod 3
)
. We consider the following cases:
(a)
m
=
8
m
for some positive integer
m
. Then
(
m
+
3
)
n
+
1

3
2
k
+
1
+
1

4
(
mod 8
)
and 3
m

0
(
mod 8
)
. So
A
is not an integer.
(b)
m
=
2
m
for some odd positive integer
m
, i.e.,
m

2
(
mod 4
)
. Then
(
m
+
3
)
n
+
1

(
2
+
3
)
n
+
1

2
(
mod 4
)
and 3
m

2
(
mod 4
)
. So
A
is odd.
(c)
m
=
4
m
for some odd positive integer
m
. Because
m
≡ −
1
(
mod 3
)
,
there exists an odd prime
p
such that
p
≡ −
1
(
mod 3
)
and
p
|
m
. Since
A
is an
integer,
0

(
m
+
3
)
n
+
1

3
2
k
+
1
+
1
(
mod
m
)
and 3
2
k
+
1
≡ −
1
(
mod
p
)
. Let
a
be a primitive root modulo
p
; let
b
be a positive
integer such that 3

a
b
(
mod
p
)
. Thus
a
(
2
k
+
1
)
b
≡ −
1
(
mod
p
)
. Note that
(
p
/
3
)
=
(

1
/
3
)
= −
1. We consider the following cases.
(i)
p

1
(
mod 4
)
. From the quadratic reciprocity law,
(

1
/
p
)
=
1, so
a
2
c
≡ −
1

a
(
2
k
+
1
)
b
(
mod
p
)
for some positive integer
c
. Therefore
b
is even and
(
3
/
p
)
=
1. Again, from the
quadratic reciprocity law,

1
=
(
3
/
p
)(
p
/
3
)
=
(

1
)
(
3

1
)(
p

1
)/
4
=
1
,
a contradiction.
(ii)
p

3
(
mod 4
)
. From the quadratic reciprocity law,
(

1
/
p
)
= −
1, so
a
2
c
+
1
≡ −
1

a
(
2
k
+
1
)
b
(
mod
p
)
for some positive integer
c
. Therefore
b
is odd and
(
3
/
p
)
= −
1. Again, from the
quadratic reciprocity law,
1
=
(
3
/
p
)(
p
/
3
)
=
(

1
)
(
3

1
)(
p

1
)/
4
= −
1
,
a contradiction.
Thus for
m
=
4
m
and
m
odd,
A
is not an integer.
From the above, we see that if
A
is an integer,
A
is odd.
Problem 9.1.4.
Prove that
2
n
+
1
has no prime factors of the form
8
k
+
7
.
(2004 Vietnamese International Mathematical Olympiad Team Selection Test)


174

Download 1,87 Mb.

Do'stlaringiz bilan baham:
1   ...   63   64   65   66   67   68   69   70   ...   125




Ma'lumotlar bazasi mualliflik huquqi bilan himoyalangan ©hozir.org 2024
ma'muriyatiga murojaat qiling

kiriting | ro'yxatdan o'tish
    Bosh sahifa
юртда тантана
Боғда битган
Бугун юртда
Эшитганлар жилманглар
Эшитмадим деманглар
битган бодомлар
Yangiariq tumani
qitish marakazi
Raqamli texnologiyalar
ilishida muhokamadan
tasdiqqa tavsiya
tavsiya etilgan
iqtisodiyot kafedrasi
steiermarkischen landesregierung
asarlaringizni yuboring
o'zingizning asarlaringizni
Iltimos faqat
faqat o'zingizning
steierm rkischen
landesregierung fachabteilung
rkischen landesregierung
hamshira loyihasi
loyihasi mavsum
faolyatining oqibatlari
asosiy adabiyotlar
fakulteti ahborot
ahborot havfsizligi
havfsizligi kafedrasi
fanidan bo’yicha
fakulteti iqtisodiyot
boshqaruv fakulteti
chiqarishda boshqaruv
ishlab chiqarishda
iqtisodiyot fakultet
multiservis tarmoqlari
fanidan asosiy
Uzbek fanidan
mavzulari potok
asosidagi multiservis
'aliyyil a'ziym
billahil 'aliyyil
illaa billahil
quvvata illaa
falah' deganida
Kompyuter savodxonligi
bo’yicha mustaqil
'alal falah'
Hayya 'alal
'alas soloh
Hayya 'alas
mavsum boyicha


yuklab olish